Absolite convergence of the beta function

Click For Summary
SUMMARY

The absolute convergence of the beta function \(\int_0^1 t^{p-1} (1-t)^{q-1}dt\) requires specific conditions on the complex parameters \(p\) and \(q\). The integral can be split into two parts: \(\int_0^{1/2} t^{p-1} (1-t)^{q-1}dt\) and \(\int_{1/2}^1 t^{p-1} (1-t)^{q-1}dt\). For convergence as \(t \to 0\), it is necessary that \(\text{Re}(p) > 0\), and for convergence as \(t \to 1\), it is essential that \(\text{Re}(q) > 0\). These conditions ensure that the integral does not encounter non-integrable neighborhoods at the endpoints.

PREREQUISITES
  • Understanding of complex analysis, particularly the behavior of complex functions near singularities.
  • Familiarity with the properties of the beta function and its integral representation.
  • Knowledge of real analysis, specifically integration techniques over bounded intervals.
  • Basic grasp of the concepts of absolute convergence in the context of improper integrals.
NEXT STEPS
  • Study the properties of the beta function and its relationship with the gamma function.
  • Learn about the implications of complex parameter conditions on integrals in complex analysis.
  • Explore examples of improper integrals and their convergence criteria.
  • Investigate the concept of singularities in integrals and their impact on convergence.
USEFUL FOR

Mathematicians, students of complex analysis, and researchers focusing on integral convergence and properties of special functions.

Ted123
Messages
428
Reaction score
0
Determine the values of the complex parameters p and q for which the beta function \int_0^1 t^{p-1} (1-t)^{q-1}dt converges absolutely.

The solution says:

Split the integral into 2 parts: \displaystyle \int_0^1 t^{p-1} (1-t)^{q-1}dt = \int_0^{1/2} t^{p-1} (1-t)^{q-1}dt + \int_{1/2}^1 t^{p-1} (1-t)^{q-1}dt

As t \to 0 |t^{p-1} (1-t)^{q-1}| \sim |t^{p-1}| \Rightarrow \text{Re}(p)>0 As t\to 1 |t^{p-1} (1-t)^{q-1}| \sim |(1-t)^{q-1}| \Rightarrow \text{Re}(q)>0

Why are these implications true?
 
Last edited:
Physics news on Phys.org
Ted123 said:
Determine the values of the complex parameters p and q for which the beta function \int_0^1 t^{p-1} (1-t)^{q-1}dt converges absolutely.

The solution says:

Split the integral into 2 parts: \displaystyle \int_0^1 t^{p-1} (1-t)^{q-1}dt = \int_0^{1/2} t^{p-1} (1-t)^{q-1}dt + \int_{1/2}^1 t^{p-1} (1-t)^{q-1}dt

As t \to 0 |t^{p-1} (1-t)^{q-1}| \sim |t^{p-1}| \Rightarrow \text{Re}(p)>0 As t\to 1 |t^{p-1} (1-t)^{q-1}| \sim |(1-t)^{q-1}| \Rightarrow \text{Re}(q)>0

Why are these implications true?

because otherwise a neighborhood of the endpoints is non-integrable?
 
Question: A clock's minute hand has length 4 and its hour hand has length 3. What is the distance between the tips at the moment when it is increasing most rapidly?(Putnam Exam Question) Answer: Making assumption that both the hands moves at constant angular velocities, the answer is ## \sqrt{7} .## But don't you think this assumption is somewhat doubtful and wrong?

Similar threads

  • · Replies 1 ·
Replies
1
Views
1K
Replies
12
Views
2K
Replies
12
Views
2K
  • · Replies 24 ·
Replies
24
Views
4K
  • · Replies 6 ·
Replies
6
Views
1K
  • · Replies 25 ·
Replies
25
Views
2K
Replies
1
Views
1K
Replies
2
Views
2K
  • · Replies 7 ·
Replies
7
Views
1K
  • · Replies 3 ·
Replies
3
Views
1K